18. Solve the equation.
m- 10/3 - 5m-9/7 = 1

Answers

Answer 1

The answer is m = -59/42

Step by step explanation is in the image below.

18. Solve The Equation.m- 10/3 - 5m-9/7 = 1

Related Questions

Please help meeeeeeeeeee

Answers

8-5=3 : This means you add three each time

f(5)=8+3=1

f(6)=11+3=14

hope this helps :)

if not snap chat has a feature in the search bar and you can get answers by that

Which represents a function? A 2-column table with 5 rows. The first column is labeled x with entries negative 10, negative 5, 0, 5, 10. The second column is labeled y with entries 84, 31.5, 4, 1.5, 24. A mapping diagram shows a relation, using arrows between domain and range for the following ordered pairs: (8, negative 4), (negative 2, 10), (negative 2, negative 7). In the domain, the number 3 does not have a relation to a number in the range. A series of ordered pairs: (4, 5), (6, negative 2), (negative 5, 0), (6, 1)

Answers

Answer:

The 1st table represents a function

Step-by-step explanation:

Just trust me on this I took the quiz on edge 202 and got it right

the graph is

x     |     Y

--------------

-5   |  -22

--------------

0    |  -2

------------

3    |  10

The store charges $1.95 for a pack of 5 pens. What is the individual price per pen ?
A - .50
B- $1.95
C- $2.56
D- .39
does anyone know or can explain ?

Answers

The answer is D, 1.95 divided by 5 equals 0.39.

If the measures of the angles in the following triangle were listed in increasing order, which measure would be second? (2x+24) ° (4x+43) ° (x^2+1) °

Answers

Answer:

x²+1 is the second

Step-by-step explanation:

2x+24=40°

4x+43=75°

x²+1=65°

(2x+24) + (4x +43) + (x²+1)=180

x²+6x+68=180

x1=-14

x2=8

since there's no negative angle, we'll use x=8

How many calories will Jacob burn in 1 minute while roller-skating????

Answers

Answer:

the answer is five

Step-by-step explanation:

first think of it like 40 over 200 then simplified down. which will get you 1/5 ,therefore five is the answer.

Resolved

1. 2(y+4) +4 (2y+2)



Answers

Answer:

10y+16

Step-by-step explanation:

Distribute and combine like terms

what is 2x + 4 = 2x + 4

Answers

Answer:

This is for x

Step-by-step explanation:

SOLVE FOR X (COMPLEX SOLUTION)

x∈C

SOLUTION STEPS

2x+4=2x+4

This is true for any x.

x∈C

SOLVE FOR X

x∈R

SOLUTION STEPS

2x+4=2x+4

This is true for any x.

x∈R

4 subtracted from twice w, the result is 20 more than w. Find w

Answers

Answer:

w = 24

I hope this helps!

W=12 as shown in pic good luck

-0.3535 is it rational or irrational​

Answers

Answer:

Rational

Step-by-step explanation:

A rational number is a number expressed as a fraction. So the number could be a decimal or percent.

Hope This Helps :)

The given decimal number is rational number.

What is the rational number?

Rational numbers are in the form of p/q, where p and q can be any integer and q ≠ 0. This means that rational numbers include natural numbers, whole numbers, integers, fractions of integers, and decimals (terminating decimals and recurring decimals).

The given decimal number is -0.3535.

Here, the given decimal number is repeated and terminating decimal

So, the given decimal is rational number

Therefore, the given decimal number is rational number.

To learn more about the rational numbers visit:

brainly.com/question/19161857.

#SPJ2

Solve for P
5/6p + 1= 3/4p -4

Answers

hii so p is equal to -60

Answer: P= -60

Step-by-step explanation:

5/6p +1=3/4p -4

5/6p=3/4p-4.       Subtract 1 from both sides

5p=9/2p-30.        Trick( if there is a fraction multiply the denominator on both sides to make it a non fraction so life is easier, as I did I first multiply by 6)

10p=9p-60.           multiply the second denominator on both sides(multiple by 2)

P=-60.                     Subtract 9p on both sides and keep the sign and you’ll get -60

Can someone solve this ASAP!!! pls thx​

Answers

Answer: just add givento number 1 and your all done bro

Step-by-step explanation:

What value of x makes the equation true?
147 = 5(1 - 7x) + 2

Answers

Answer: X = -4

Step-by-step explanation:

first you need to solve, so do what needs to be done in the equation, which is just multiply and add

147 = 5 - 35x + 2

147= 7 -35x

get rid of the numbers around x, which would be subtracting 7 from both sides

140 = -35x

x = -4

Which of these would not produce a representative sample that determines the favorite sport of the students at the local high school?

Answers

Answer:

The answer is ask ten students wearing football jersey each day for a week

Step-by-step explanation:

Answer:

it its the pizza one because people could be bias and that would not give a very acturate answer

Step-by-step explanation:

D
Question 3
Write the phrase as an expression.
12 more than the quotient of a number k and 5


Answers

Answer:

12+k/5

Quotient is a quantity produced by the division of two numbers.

Find the value of each variable (x in this case)

Answers

2x and 4x + 108 are supplementary angles and that means they add up to 180 degrees.

2x + 4x + 108 = 180

Combine like terms

6x + 108 = 180

Subtract 108 from both sides.

6x = 72

Divide both sides by 6

x = 12

Find the slope
x+3=0

Answers

Answer: undefined

You would subtract 3 from both sides to get x= -3 which would be a vertical line meaning it has no slope

Find the value of x that will make L || M

Answers

Answer:

The value of x = 7 to make lines L and M parallel.

Use the image below to solve for x.

Answers

Answer:

x will equal 61 degrees

Step-by-step explanation:

The total degree of a pentagon is 540. You subtract all the angles you have (138, 144, 107, and 90) and that will get your sum.

Why is it imprecise to say that the equation 25 x 2 + 27 + 0 has no solution? Find all solutions of the equation.

Answers

Answer:

77

Step-by-step explanation:

77 is the only correct answer

Find the distance between the two points. (round to the nearest hundredths)



distance:

Answers

Answer:

d = 5.10

Step-by-step explanation:

The distance between two points is given by

d = sqrt( ( y2-y1)^2 + ( x2-x1)^2 )  where ( x1,y1) and ( x2,y2) are the two points

d = sqrt( ( 5-6)^2 + (8-3)^2 )  

d = sqrt( ( -1)^2 + (5)^2 )  

d = sqrt(  1+25)

d = sqrt(26)

d = 5.099019514

Rounding to the nearest hundredth

d = 5.10

Answer:

[tex]d=\sqrt{26}\approx5.10[/tex]

Step-by-step explanation:

To find the distance between any two points, we can use the distance formula:

[tex]d=\sqrt{(x_2-x_1)^2+(y_2-y_1)^2[/tex]

We can see that Point R is at (3,6), and Point E is at (8,5).

Let's let (3,6) be (x₁, y₁) and let's let (8,5) be (x₂, y₂). So, substituting them into our formula, we'll get:

[tex]d=\sqrt{(8-3)^2+(5-6)^2[/tex]

Subtract:

[tex]d=\sqrt{(5)^2+(-1)^2[/tex]

Square:

[tex]d=\sqrt{25+1}[/tex]

Add:

[tex]d=\sqrt{26}[/tex]

Approximate. Use a calculator:

[tex]d=\sqrt{26}\approx5.10[/tex]

So, the distance between RE is approximately 5.1 units.

And we're done!

18 over 30 in its simplest form

Answers

Answer:

3/5

Step-by-step explanation:

18/ 30 can be simplified by diving it by 3

6/10 simplified again by 2

3/5

Answer: 3/5

Step-by-step explanation:

could someone help me with this, I have a few minds left. thanks!! Also marking as brainiest.

Answers

10 is in between the perfect squares of 3 and 4 (because 3² is 9 and 4² is 16)

so √10 is located between √9 and √16

∠A and ∠ B ∠B are complementary angles. If m ∠ A = ( 2 x − 10 ) ∘ ∠A=(2x−10) ∘ and m ∠ B = ( x − 2 ) ∘ ∠B=(x−2) ∘ , then find the measure of ∠ A ∠A.

Answers

Answer:

Angle A = 58 degrees

Angle B = 32 degrees

Step-by-step explanation:

First of all, we will need to know that complementary angles are angles that sum up to 90 degrees.

This means that if we add the values of angle A and angle B, they will both sum up to 90 degrees.

i.e (2x-10) + (x-2) = 90

3x -12 = 90

3x = 102

x = 34 degrees

We can substitute this value of x into the equations for angles A and B

A = (2 X (34) - 10 ) = 58 degrees

B =  ( 34 - 2) = 32 degrees

Therefore, angles A and B are 54 and 32 degrees respectively.

Check: We can add the two angles to see if we get 90 degrees. 58 + 32 = 90 degrees. This shows that the two angles are complementary and our answer is correct.

if 18 more than twice a number is -10,what is the number?​

Answers

Answer:

-14

Step-by-step explanation:

Let the number be x

[tex]2x + 18 = - 10 \\ 2x = - 28 \\ x = - 14[/tex]

2x + 18= -10
2x. -18= -10 -18
2x= -28
X=-14

6. Evaluate the function at each specified value of the independent variable and simplify. (If an answer is undefined, enter UNDEFINED.) 7. Evaluate the function at each specified value of the independent variable and simplify. (If an answer is undefined, enter UNDEFINED.)

Answers

Complete Question

The complete question is shown on the first uploaded image

Answer:

6a

    [tex]V(3) =  113.112[/tex]

6b

     [tex]V(\frac{3}{2})  = 14.139  [/tex]

6c

   [tex]V(2r) =  33.514r^3[/tex]

7a

  [tex] h(5) = 0 [/tex]

7b

    [tex] h(t) = -5.76 [/tex]

7c

[tex] h(x+5) = x(x+5)[/tex]

Step-by-step explanation:

Considering the question 6

The function given is [tex]V(r) = \frac{4}{3} \pi r^3[/tex]

For  V(3) we have  

        [tex]V(3) =  \frac{4}{3} \pi 3^3[/tex]

       [tex]V(3) =  \frac{4}{3} * 3.142 * 3^3[/tex]

       [tex]V(3) =  113.112[/tex]

For  [tex]V(\frac{3}{2})[/tex]

        [tex]V(\frac{3}{2})  =  \frac{4}{3} *  \pi *   (\frac{3}{2})^3 [/tex]

        [tex]V(\frac{3}{2})  =  \frac{4}{3} * 3.142 *   (\frac{3}{2})^3 [/tex]

        [tex]V(\frac{3}{2})  = 14.139  [/tex]

For   V(2r)

            [tex]V(2r) =  \frac{4}{3} \pi (2r)^3[/tex]

           [tex]V(2r) =  \frac{4}{3} * 3.142*8r^3[/tex]

           [tex]V(2r) =  33.514r^3[/tex]

Considering the question 7

     The function given is  [tex] h(t) = t^2 - 5t [/tex]

For  h(5)

          [tex] h(5) = 5^2 - 5(5) [/tex]    

           [tex] h(5) = 0 [/tex]  

For  h(1.8)

            [tex] h(t) = 1.8^2 - 5(1.8) [/tex]

            [tex] h(t) = -5.76 [/tex]

For  h(x+ 5)

              [tex] h(x+5) = (x+5)^2 - 5(x+5) [/tex]

               [tex] h(x+5) = x^2 + 25+10x-5x-25[/tex]

             [tex] h(x+5) = x(x+5)[/tex]

half a number decreased by three is equal to 38 less than four times the number

Answers

Answer:

10

Step-by-step explanation:

let the unknown number be x

so half of x decreased by 3 is

1/2x -3

It is equal to 38 less than 4 times the number which is x; 4x-37

THEREFORE;

½x-3 =4x -38

x-6/2 = 4x-38

cross-multiply

x-6 =8x -76

x-8x = -76+6

-7x = -70

Divide both sides by -7

So x =10

Help I really don't know how to do this I need help !!!!!!

Answers

Answer:

It would be C for it is the lowest.

Step-by-step explanation:

Since all of these numbers are fractions we can divide them by the denominator and see which one would be the lowest.  If we do that we will end up getting C.

A - 0.5

B - 0.5

C - 0.4

D - 0.45

Which recursive formula can be used to generate the sequence shown, where f(1) = 9.6 and n > 1? 9.6, –4.8, 2.4, –1.2, 0.6, ...​

Answers

Answer:

whoa slow down one by on pls its hard for me 2 answer all that at once

Step-by-step explanation:

of the function.
16. y = 3x + 4
Domain:0, 5, 7, 10
find the range

Answers

Answer:

all is pictured and shown

Taki's are originally $3.25 a bag. You have a 25% off coupon. What
is the final price of the Takis?

Answers

Answer:

3.25 * .75 = $2.44

Step-by-step explanation:

Answer:

Your answer should be $2.44 or around there

Step-by-step explanation:

Other Questions
Weldon bought a pair of hiking boots that cost $58.50 before tax. If an 8.5% tax is added to Weldon's total, how much did he spend in total on the hiking boots? Find the sum ( u2-6uv+5v2)+3u2+2uv Lucy is going to the grocery store. She will buy 5 items. Every item she will buy is $2. Afterward, she will buy herself a milk shake for $3. How much will she spend at the store? Where did the Nat Turner rebellion take place? 0.45 repeating a whole number? Natural number? Real number? Whats this polynomial in standard form heiyhbhjbhvhjbisdvhbisbvdisuvbikhbfikjbivkbunklsfjubkijnbikbujikhbjhb nb jbbkkjbkjj khbhbokm hjkmn bvfcdxrtfyguhijkmjn bvcgxd rdngtrt6yh7 ujjnumhibhn vgcfvdsbgv hbjmg, ngvftngmyhnjm bnbhjkljm Escribe tres versos heptaslabos de rima asonante (AAA) que describan tu pelcula favorita(Puede ser su pelcula favorita) if it is 5:30 pm what time would it be a half and hour later 15x + 3 + 2x + 9r - (8 - r) Chlorine could combine with _ to gain the extra electron it needs to stabilize its outer shell outer shell.A. heliumB. sulfurC. hydrogenD. none of these Amtrail trains provide efficient, nonstop transportation between Los Angeles and San Diego. Train A leaves Los Angeles headed toward San Diego at the same time that Train B leaves San Diego headed for Los Angeles, traveling on parallel tracks. Train A travels at a constant speed of 84 miles per hour. Train B travels at a constant speed of 92 miles per hour. The two stations are 132 miles apart. How long after they leave their respective stations do the trains meet Ashley wants to know the oldest form of both water purification and waste disposal. Help Ashley determine the methods. The oldest method of water purification is . The oldest method of waste disposal is . how many animals do u see What might happen to the organisms in the food web below if the number of phytoplankton and vegetation drasticallydecreased ? A. the small fish and wading birds would be affected B. Only the small fish would be affected C. The small fish, bivalves, and invertebrates would be affected D. The entire food web would be affected What is 4 divided by 6 Name (3) unalienable Rights?? Help me please ASAP!! :( (1 point) Consider the system2x + y = 3-8x + ay = -11Find a value of a so that the system is inconsistent. If there is no such value, write none.a = Jose is taking a multiple-choice test. After he reads the questions, he looks at the potentialanswers to see which ones make sense and seem correct. In regard to memory, this is an exampleof